Obstetrics Flashcards

1
Q

What needs to be discussed with a women before she gets pregnant?

HINT: preconception counseling.

A
  • past medical history: optimize medical illnesses and necessary medications prior to pregnancy
  • supplementation
    • folic acid: encourage diet rich in folic acid and supplement 8-12 wk preconception until end
      of T1 to prevent NTDs
      • 0.4-1 mg daily in all women; 5 mg if previous NTD, antiepileptic medications, DM, or
        BMI >35 kg/m 2
    • iron supplementation, prenatal vitamins
  • risk modification
    • lifestyle: balanced nutrition and physical fitness
    • medications: patients with chronic diseases should discuss whether their medications may be teratogenic prior to conception so they may be adjusted; it is not advised to stop medications abruptly when becoming pregnant
    • infection screening: rubella, HBsAg, VDRL, Pap smear, gonorrhea/chlamydia, HIV
    • genetic testing as appropriate for high risk groups; consider genetics referral in known carriers, recurrent pregnancy loss/stillbirth, family members with
      developmental delay or birth anomalies
    • social: alcohol, smoking, drug use, domestic violence
How well did you know this?
1
Not at all
2
3
4
5
Perfectly
2
Q

Genetic counselling.

Downs syndrome

Incidence, mode of inheritence, population at risk, screening and Dx, clinical manifestations and natural Hx.

A
  • Incidence – most common chromosomal abnormality among live births; 1 per 1,000
  • Underlying abnormality – three copies of chromosome 21
  • Risk factors – high maternal age, previous affected baby (risk increased 1%), balanced chromosomal translocation in parents (6% of cases)
  • Screening and diagnosis
    • 1st trimester – abnormalities on U/S (thickened nuchal translucency, structural abnormalities, absent or shortened nasal bone, tricuspid regurgitation), high ß-hCG, low PAPP-A
    • 2nd trimester – triple test (high ß-hCG, low AFP, low unconjugated oestriol (UE3))
  • Clinical manifestations – globally delayed development, characteristic facial appearance, significant risk of specific malformations (atrioventricular septal defect, duodenal atresia)
  • Natural history – reduced life expectancy, typically poor immune system, predisposition to several lateonset disorders including hypothyroidism, acute leukaemias and Alzheimer’s disease
How well did you know this?
1
Not at all
2
3
4
5
Perfectly
3
Q

Genetic counselling.

Cystic fibrosis.

Incidence, mode of inheritence, population at risk, screening and Dx, clinical manifestations and natural Hx.

A
  • Incidence: 1 per 3,000 newborns; about 1 in 25 are carriers
  • Underlying abnormality: mutations in both copies of the CFTR gene, most common mutation is F508
  • Mode of inheritance: autosomal recessive
  • Populations at risk: Mediterranean, Finnish, Caucasian, or FHx
  • Screening and diagnosis: sweat test and genetic testing (inc. Guthrie)
  • Clinical manifestations:
    • salty-tasting skin
    • poor growth/weight gain despite normal food intake
    • accumulation of thick sticky mucous: frequent chest infections, coughing, shortness of breath
    • male infertility (congenital absence of the vas deferens)
    • bowel obstruction due to meconium ileus in newborns
  • Natural history: prognosis has improved with median survival ~50yo
How well did you know this?
1
Not at all
2
3
4
5
Perfectly
4
Q

Genetic counselling.

Thalassaemia

Incidence, mode of inheritence, population at risk, screening and Dx, and clinical manifestations.

A
  • Incidence: at least 5.2% of the world population carry a significant variant
  • Underlying abnormality: defects in either the α or ß globin chains in adult haemoglobin
  • Mode of inheritance: autosomal recessive
  • Populations at risk: Mediterranean, South East Asian, Western Pacific, African, Middle Eastern, Caribbean, South American
  • Screening and diagnosis: FBC (MCV and MCH), Hb electrophoresis
  • Clinical manifestations:
    • iron overload (from disease or frequent blood transfusions)
    • infection (especially if spleen has been removed)
    • bone deformities
    • enlarged spleen
    • slowed growth rates (from anaemia)
    • delayed puberty
    • heart problems e.g. CHF and arrhythmias
How well did you know this?
1
Not at all
2
3
4
5
Perfectly
5
Q

Describe early pregnancy physiology with respect to fertilisation, implantation and placental development.

A
  • Fertilisation - Penetration of the zona pellucida triggers three events:
    • Egg cell membrane depolarises, preventing membrane fusion with additional sperm.
    • Cortical granules just beneath the egg cell membrane fuse with the membrane and release their contents, hardening the zona and impairing the ability of sperm to bind to it.
    • Resumption of the third meiotic division of the egg
  • Implantation
    • The cleaving zygote floats towards the uterus approximately 1wk and enters at blastocyst stage
    • After 2d in the uterus the blastocyst will lose the zona pellucida, and trophoblast cells make direct contact with the uterine luminal epithelial cells allowing for implantation, eroding the lining
    • Endometrium undergoes decidualisation and heals over the conceptus
  • Placental development
    • At the blastocyst stage, the embryo is characterised by a fluid-filled cavity (the blastocele) surrounded by a layer of trophectoderm cells that will develop into the placenta
    • During implantation, trophectoderm cells begin to differentiate into cellular subtypes that will characterise the mature placenta.
How well did you know this?
1
Not at all
2
3
4
5
Perfectly
6
Q

Describe the physiological changes that occur in pregnancy.

Cardiovascular.

A
  • Hyperdynamic circulation
  • Increased CO, HR, and blood volume
  • Decreased BP due to decreased PVR
  • Enlarging uterus compresses IVC and pelvic veins
  • Decreased venous return leads to risk of hypotension
  • Increased venous pressure leads to risk of varicose veins, hemorrhoids, leg edema
How well did you know this?
1
Not at all
2
3
4
5
Perfectly
7
Q

Describe the physiological changes that occur in pregnancy.

Renal

A
  • Increased urinary frequency due to increased total urinary output
  • Increased incidence of UTI and pyelonephritis due to urinary stasis
  • Glycosuria that can be physiologic especially in the 3rd trimester
  • Ureters and renal pelvis dilation (R>L) due to progesterone-induced smooth muscle relaxation and
    uterine enlargement
  • Increased CO and thus increased GFR leads to decreased creatinine (normal in pregnancy 0.4-0.5 mg/dL), uric acid, and BUN
How well did you know this?
1
Not at all
2
3
4
5
Perfectly
8
Q

Describe the physiological changes that occur in pregnancy.

Respiratory

A
  • Increased incidence of nasal congestion and epistaxis
  • Increased O2 consumption to meet increased metabolic requirements
  • Elevated diaphragm (i.e. patient appears more “barrel-chested”)
  • Increased minute ventilation leads to decreased CO2 resulting in mild respiratory alkalosis that helps
    CO2 diffuse across the placenta from fetal to maternal circulation
  • No change in VC and FEV1
  • Decreased TLC, FRC, and RV
How well did you know this?
1
Not at all
2
3
4
5
Perfectly
9
Q

Describe the physiological changes that occur in pregnancy.

HAematological

A
  • Hemodilution causes physiologic anemia and apparent decrease in hemoglobin and hematocrit
  • Increased leukocyte count but impaired function leads to improvement in autoimmune diseases
  • Gestational thrombocytopenia: mild (platelets >70,000/µL) and asymptomatic, normalizes within 2-12 wk following delivery
  • Hypercoagulable state: increased risk of DVT and PE but also decreased bleeding at delivery
How well did you know this?
1
Not at all
2
3
4
5
Perfectly
10
Q

Describe the minor complications of pregnancy and their management.

A
  • Itching: common; liver complications in pregnancy often present with itching and so sclerae checked for jaundice and LFTs and bile acids to be assessed
  • Pelvic girdle pain: common, cured after delivery; physiotherapy, corsets, analgesics and even crutches may be used
  • Abdominal pain: universal to some degree; check for common medical and surgical problems such as appendicitis, and pancreatitis; UTIs and fibroids may cause pain in pregnancy
  • Heartburn: affects 70%, most marked in supine position; extra pillows are helpful, antacids are not contraindicated and ranitidine can be used in severe cases; pre-eclampsia can present with epigastric pain
  • Backache: almost universal and may cause sciatica, most cases resolve after delivery; physiotherapy, advice on posture and lifting, a firm mattress and a corset may all help
  • Constipation: common and exacerbated by oral iron; high fibre intake required, stool softeners used if this fails
  • Ankle oedema: common, worse towards the end of pregnancy; benign oedema helped by raising the foot of the bed at night, diuretics should not be given; oedema is an unreliable sign of pre-eclampsia, but sudden increase in oedema warrants careful assessment and follow-up BP and urinalysis
  • Leg cramps: affect 30%, treatments unproven
  • Carpal tunnel syndrome: due to fluid retention compressing the median nerve, seldom severe and usually temporary; wrist splints may help
  • Vaginitis – due to candidiasis, common in pregnancy and more difficult to treat; imidazole vaginal pessaries (e.g. clotrimazole) used for symptomatic infection (do not use oral treatments due to increased risk of birth defects)
  • Tiredness: almost universal and often incorrectly attributed to anaemia
How well did you know this?
1
Not at all
2
3
4
5
Perfectly
11
Q

Discuss the following topics in pregnancy:

  • Diet and exercise
  • weight gain
  • alcohol
  • smoking
  • Dental check-up
  • Coitus
  • Avoidance of infection
  • Work
  • travel
  • seat belts
A
  • Diet and exercise – well balanced diet, exercise advised (avoid heavy contact sports)
  • Weight gain in pregnancy – varies by body mass, for normal weight recommended range of 11.3-15.9kg, for obese recommended range of 5.0-9.0kg
  • Alcohol – no amount of alcohol is safe in pregnancy, it increases incidence of abortion, stillbirth and congenital anomalies
  • Smoking – increased risk of decreased birth weight, placenta praevia/abruption, spontaneous abortion, pre-term labour and stillbirth; nicotine replacement therapy may be used
  • Dental check-up advised
  • Coitus – not contraindicated except when in placenta praevia or ruptured membranes
  • Avoidance of infection – listeriosis avoided by drinking only pasteurised or UHT milk, by avoiding soft and blue cheese, pate and uncooked or partially cooked ready prepared food; salmonella avoided by cooking eggs or poultry well
  • Work – strenuous work, extended hours and shift work during pregnancy may be associated with greater risk of low birth weight, prematurity, and spontaneous abortion
  • Travel – not harmful, but stress related to travel may be associated with preterm labour; air travel is acceptable in second trimester; airline cutoff for travel is 36-38 wk gestation depending on the airline to avoid giving birth on the plane
  • Other – when driving a seatbelt should be worn above and below the ‘bump’, sleeping should be in the left lateral position
How well did you know this?
1
Not at all
2
3
4
5
Perfectly
12
Q

How can pregnancy be diagnosed?

A
  • Menstrual history – date of onset of last menses, duration, flow and frequency
    • Items that may confuse diagnosis – atypical last menstrual period, contraceptive use, irregular menses
    • Symptoms – amenorrhoea, nausea and/or vomiting, breast tenderness, urinary frequency, fatigue
  • Physical signs
    • Goodell’s sign – softening of the cervix (4-6wk)
    • Chadwick’s sign – bluish discolouration of the cervix and vagina due to pelvic vasculature enlargement (6wk)
    • Hegar’s sign – softening of the cervical isthmus (6-8wk)
    • Uterine enlargement on bimanual examination, uterus may be palpable low in the adomen (~12wk)
    • Breast engorgement
  • Investigations
    • ß-hCG – hormone produced by placental trophoblastic cells that maintain the corpus luteum
      • Positive in serum 9 d post-conception, positive in urine 28 d after first day of LMP
      • Plasma levels double every 1-2 d, peak at 8-10 wk, then fall to a plateau until delivery
    • U/S
      • Transvaginal – at 5wk gestational sac visible, at 6wk foetal pole seen, at 7-8wk foetal heart tones
      • Transabdominal – at 6-8 wk intrauterine pregnancy visible
How well did you know this?
1
Not at all
2
3
4
5
Perfectly
13
Q

What methods are used in pre-natal testing for congenital abnoralities?

A
  • Maternal blood testing
    • AFP, ß-hCG, pregnancy-associated plasma protein A (PAPP-A), oestriol and inhibin A
    • Can be integrated with other risk factors such as maternal age and ultrasound measurements (e.g. nuchal translucency) to screen for the trisomies 21, 18 and 13
  • Ultrasound
    • Screening – nuchal translucency (the space between skin and soft tissue overlying the cervical spine, the larger it is the higher the risk) at 11-14wk
    • To aid other diagnostic tests – amniocentesis and chorionic villus sampling (CVS) under U/S guidance
    • As a diagnostic test – structural abnormalities usually diagnosed at the ‘anomaly scan’
  • 3-D/4-D ultrasound – can allow better evaluation of certain abnormalities
  • Amniocentesis
    • U/S trans-abdominal extraction of amniotic fluid, safest performed from 15wk gestation
    • Allows prenatal diagnosis of chromosomal abnormalities, some infections such as CMV and toxoplasmosis, and inherited disorders such as sick-cell anaemia, thalassaemia and cystic fibrosis
    • Risk of miscarriage in 1%
  • Chorionic villus sampling
    • Biopsy of foetal-derived chorion using a trans-abdominal need or transcervical
      catheter at 10-12wk
    • Allows earlier testing than amniocentesis, so termination can occur if necessary
    • Used to diagnosis chromosomal and single gene disorders
    • Higher risk of miscarriage than amniocentesis because it is performed earlier
How well did you know this?
1
Not at all
2
3
4
5
Perfectly
14
Q

What are some teratogenic agents?

A
  • Alcohol, smoking and illegal drugs
    • Alcohol: FAS
    • Cocaine: microcephaly, growth retardation, prematurity, abruptio placentae
    • smoking: assist/encourage to reduce or quit smoking
      • increased risk of: decreased birth weight, placenta previa/abruption, spontaneous abortion, preterm labor, stillbirth
  • Medications
    • Phenytoin (cat D) - Fetal hydantoin syndrome in 5-10% (IUGR, mental retardation, facial dysmorphogenesis, congenital anomalies)
    • Retinoid - CNS, craniofacial, cardiac, and thymic anomalies
    • ACE inhibitors (Cat D) - Fetal renal defects, IUGR, oligohydramnios
    • Lithium - Ebstein’s cardiac anomaly, goitre, hyponatraemia
    • Valproic acid (cat D)
    • Warfarin - increased incidence of spontaneous abortion, stillbirth, prematurity, IUGR, fetal warfarin syndrome (nasal hypoplasia, epiphyseal stippling, optic atrophy, mental retardation, intracranial haemorrhage)
    • Thalidomide
    • Tetracyclines (Cat D) - Stains infant’s teeth, may affect long bone development
  • Infections
    • Syphilis
    • Toxoplasmosis
    • Chickenpox
    • Rubella: cataracts
  • Chemicals
    • lead: miscarriages and stillbirth
    • mercury: mental retardation
  • Ionising radiation: >5 rads needed for miscarriage, >20-30 rads needed for malformations.
How well did you know this?
1
Not at all
2
3
4
5
Perfectly
15
Q

Describe the schedule of routine antenatal visits.

A
  • Initial vist: FBC, blood group + red cell antibpdy screen, Hep B, rubella IgG, syphilis serology, HIV serology, HCV antibodies, Varicella IgG, Urine culture
  • 12-14 wk: US dating, nuchal transluency, nasal bone (T21), ductus venous flow (T21, cardiac, TTS), number of fetuses.
  • 20wk – an ultrasound should be offered, this ‘anomaly scan’ enables detection of most structural foetal abnormalities, although reported success rates vary widely
  • 26wk – OGTT, FBC, antibody screen (if Rhesus negative)
  • 30wk - Midwife
  • 33wk - Anti-D, US if placentta is low lying at 20 weeks
  • 36wk – low vaginal swab for Group B Streptococcus
  • 39wk -
  • 41wk – assessment for induction of labour for post-dates
How well did you know this?
1
Not at all
2
3
4
5
Perfectly
16
Q

Outline the physiology of amniotic fluid volume?

A
  • Early gestation
    • Volume increases prior to the transition from embryo to foetus (i.e. 10wk gestation)
    • Likely derived from the foetal surface of the placenta, transport from the maternal compartment across the amnion (transmembranous), secretions from the surface of the body of the embryo
  • Mid gestation – transition from embryo to foetus
    • Foetal urine begins to enter the amniotic sac, daily volumes quite small
    • Foetal lungs begin to secrete liquid into the AF at this time
  • Late gestation
    • Production – foetal urine and foetal lung liquid, minor secretion from the oral-nasal cavities
    • Clearance – foetal swallowing, intramembranous pathway, transmembranous pathway (minor)
How well did you know this?
1
Not at all
2
3
4
5
Perfectly
17
Q

Define intrauterine growth restriction?

A

infant weight <10th percentile for GA or <2,500 g

How well did you know this?
1
Not at all
2
3
4
5
Perfectly
18
Q

What are the causes of intrauterine growth restrition?

A
  • maternal causes
    • malnutrition, smoking, drug abuse, alcoholism, cyanotic heart disease, type 1 DM, SLE, pulmonary insufficiency, previous IUGR
  • maternal-fetal
    • any disease causing placental insufficiency
    • includes gestational HTN, chronic HTN, chronic renal insufficiency, gross placental morphological abnormalities (infarction, hemangiomas)
  • fetal causes:
    • TORCH infections, multiple gestation, congenital anomalies
How well did you know this?
1
Not at all
2
3
4
5
Perfectly
19
Q

Describe the clinical features of symmetric and asymmetric intrauterine growth restrition? And there complications.

A
  • Symmetric/type I (20%): occurs early in pregnancy
    • inadequate growth of head and body
    • head:abdomen ratio may be normal (>1 up to 32 wk; =1 at 32-34 wk; <1 after 34 wk GA)
    • usually associated with congenital anomalies or TORCH infections
  • Asymmetric/type II (80%): occurs late in pregnancy
    • brain is spared, therefore head:abdomen ratio increased
    • usually associated with placental insufficiency
    • more favorable prognosis than type I
  • complications
    • prone to meconium aspiration, asphyxia, polycythemia, hypoglycemia, and mental retardation
    • greater risk of perinatal morbidity and mortality
How well did you know this?
1
Not at all
2
3
4
5
Perfectly
20
Q

Describe the role, methods, benefits and limitations to antenatal fetal monitoring.

A
  • Routine pregnancy care – cornerstone of the identification of the small or compromised foetus is serial measurement of the symphysis fundal height and other aspects of antenatal visits
  • US assessment of foetal growth
    • Three factors help differentiate between the healthy small foetus and the ‘growth-restricted’ foetus
      • Rate of growth can be determined by 2 scans at least 2wk apart
      • The foetal abdomen will often stop enlarging before the head
      • Able to assess actual vs expected growth
    • Benefits – safe and useful in confirming consistent growth in high-risk and multiple pregnancies
    • Limitations – ‘one-off’ scans later in pregnancy are of limited benefit in low-risk pregnancies, inaccurate measurements are common and misleading
  • Doppler umbilical artery waveforms
    • Used to measure velocity waveforms in the umbilical arteries, evidence of high resistance circulation suggests placental dysfunction
    • Benefits – allows identification of which small foetuses and growth restricted and therefore compromised, absence of flow usually pre-dated CTG abnormalities
    • Limitations – not useful screening tool in low-risk pregnancies, less effective at identifying normalweight but compromised foetus
  • Doppler waveforms of the foetal circulation
    • With foetal compromise the middle cerebral artery often develops a low-resistance flow, velocity of flow increases with foetal anaemia
    • Benefits: restricted to high risk-pregnancy and specific situations, contributes to decision-making
    • Limitation: routine use does not reduce perinatal morbidity or mortality
  • Cardiotocography (CTG)
    • The foetal heart is recorded electronically for up to an hour, accelerations and variability >5bpm should be present, decelerations absent and the rate in the range of 110-160
    • Benefits – of benefit in delaying delivery of chronically compromised premature foetuses
    • Limitation – of no use alone, to be useful in high-risk pregnancy it needs to be performed every day
      (as can only predict that foetus will not die in next 24h)
    • If abnormalities, perform biophysical profile
  • US assessment of biophysical profile / amniotic fluid volume
    • Four variables (limb movements, tone, breathing movements and liquor volume) and ‘scored’, CTG may be added to the score; low score suggests serious compromise
    • Benefits – useful in high-risk pregnancy where CTG or Doppler give equivocal results
    • Limitations – it is time consuming and is of little use in the low-risk pregnancy
  • Kick chart – mother records number of individual movements that she experiences every day, however compromised foetuses stop moving only short before death, and routine counting is of little use
How well did you know this?
1
Not at all
2
3
4
5
Perfectly
21
Q

What are the DDx for incorrect uterine size for dates?

A
  • Inaccurate dates
  • Maternal: DM
  • Maternal-fetal: polyhydramnios, oligohydramnios
  • Fetal: abnormal karyotype, IUGR, macrosomia, fetal anomaly, abnormal lie, multiple gestation.
How well did you know this?
1
Not at all
2
3
4
5
Perfectly
22
Q

What are the indications for termination of pregnancy?

A

inability to carry a pregnancy to term due to medical or social reasons (including patient preference)

How well did you know this?
1
Not at all
2
3
4
5
Perfectly
23
Q

Describe the options for termination of pregnancy?

A
  • medical
    • <9 wk: methotrexate + misoprostol
    • >12 wk: prostaglandins (intra- or extra-amniotically or IM) or misoprostol
  • surgical
    • <12 wk: dilatation + vacuum aspiration ± curettage
    • >12 wk: dilatation and evacuation, early induction of labor
    • common complications: pain or discomfort
    • less common complications: hemorrhage, perforation of uterus, laceration of cervix, risk of infertility, infection/endometritis, Asherman’s syndrome (adhesions within the endometrial cavity causing amenorrhea/infertility), retained products of conception
How well did you know this?
1
Not at all
2
3
4
5
Perfectly
24
Q

What are the types of hypertension in pregnancy? and their defintions.

A
  • Pre-existing HTN
    • HTN (>140/90) prior to 20 wk GA, persisting >7 wk postpartum or the woman is already on antihypertensive medication
      • Either primary or secondary (to renal or other disease)
      • Increased risk (6-fold) of ‘superimposed’ pre-eclampsia
  • Pregnancy-induced HTN: sBP >140 or dBP >90 developing after 20th wk GA in a woman known to be normotensive before pregnancy
    • Gestational HTN - new hypertension without proteinuria
    • Pre-eclampsia
      • hypertension and proteinuria (>0.3g/24h) appear in the second half of pregnancy, often with oedema; eclampsia (epileptiform seizures) a complication
How well did you know this?
1
Not at all
2
3
4
5
Perfectly
25
Q

Describe the pathophysiology of pre-eclampsia?

A
  • Endothelial cell damage in the placenta in association with an exaggerated maternal inflammatory response (due to incomplete trophoblastic invasion of spiral arterioles) causes ischaemia, which leads to vasospasm, increased capillary permeability and clotting dysfunction
  • This leads to increased vascular resistance (hypertension), increased vascular permeability (proteinuria), reduced placental blood flow (IUGR) and reduced cerebral perfusion (eclampsia)
How well did you know this?
1
Not at all
2
3
4
5
Perfectly
26
Q

What are the risk factors for pre-eclampsia?

A
  • nulliparity
  • preeclampsia in a previous pregnancy
  • age >40 yr or <18 yr
  • FHx of preeclampsia
  • chronic HTN
  • chronic renal disease
  • antiphospholipid antibody syndrome or inherited thrombophilia
  • vascular or connective tissue disease
  • DM (pre-gestational and gestational)
  • high BMI
  • hydrops fetalis
  • unexplained fetal growth restriction
  • abruptio placentae
  • there is a potential for further deterioration to severe preeclampsia
  • the adverse conditions are many and include both maternal and fetal issues
How well did you know this?
1
Not at all
2
3
4
5
Perfectly
27
Q

What are the macroscopic changes that occur to a placenta in pre-eclampsia?

A
  • Small placenta – looks like a cupcake
  • Serial slices show cream-white areas of infarction which involve more
    than 25% of the placental parenchyma
  • Retroplacental haematomas are more common
How well did you know this?
1
Not at all
2
3
4
5
Perfectly
28
Q

Describe the microscopic features of pre-eclampsia?

A
  • Lack of conversion in decidual vessels
  • Aggregates of foamy macrophages
  • Fibrinoid necrosis-atherosis in decidual arteries
  • Coagulative necrosis in areas of infarction
  • Accelerated maturation in some villi
  • Occasional nucleated RBCs
How well did you know this?
1
Not at all
2
3
4
5
Perfectly
29
Q

What Ix need to be ordered for a patient with pre-eclampsia?

A
  • Bloods:
    • FBC and peripheral smear: microangiopathic haemolytic anaemia (HELLP), schistocytes on peripheral smear, thrombocytopaenia<100,000, haemoconcentration in severe cases
    • LDH
    • LFT: transaminase levels elevated from hepatocellular injury and in HELLP syndrome
    • INR and aPTT
    • UEC: serum creatinine levels elevated due to decreased intravascular volume and decreased GFR
    • Urine (dip ± 24 h collection or protein/creatinine ratio)
    • Uric acid: hyperuricaemia one of the earliest lab manifestations, serial levels indicate progression
  • Imaging:
    • US: to assess foetal well-being and evaulate IUGR
    • If US is abnormal: umbilical artery Doppler and cardiotocography to evaluate foetal well-being
How well did you know this?
1
Not at all
2
3
4
5
Perfectly
30
Q

What are the ominous symptoms of HTN in pregnancy?

A
  • RUQ pain
  • headache
  • visual disturbances
How well did you know this?
1
Not at all
2
3
4
5
Perfectly
31
Q

What are the clinical features of pre-eclampsia?

A
  • History:
    • usually asymptomatic
    • headache, drowsiness, visual disturbances, nausea/vomiting or epigastric pain may occur at a late stage
  • Examination:
    • HTN usually the first sign
    • oedema (massive, not postural, or of sudden onset)
    • epigastric tenderness
    • urine dipstick (1+ possibly significant, ≥2+ likely significant)
How well did you know this?
1
Not at all
2
3
4
5
Perfectly
32
Q

What is the management of a patient with pre-eclampsia?

A
  • Without proteinuria and mild/moderate hypertension – managed as outpatients, BP and urinalysis repeated twice weekly and U/S performed every 2-4wk unless foetal compromise
  • Severe hypertension or proteinuria – admit to hospital
    • Drugs
      • Anti-hypertensives if BP>150/100mmHg – labetalol (1st), methyldopa, nifedipine, hydralazine
      • Magnesium sulphate – used for treatment and, if severe, prevention of eclampsia
      • Steroids – used to promote foetal pulmonary maturity if gestation <34wk
    • Timing of delivery – mild pre-eclampsia required delivery by 37wk, moderate of severe preeclampsia requires delivery if gestation exceeds 34-36wk, clinical deterioration prompts delivery
    • Conduct of delivery – caesarean section if before 34wk, induction after 34wk
    • Post-natal care – usually takes at least 24h for condition to improve post-delivery, so monitor bloods (FBC, LFT, EUC) and fluid balance, treatment for BP may be required for several weeks.
How well did you know this?
1
Not at all
2
3
4
5
Perfectly
33
Q

What are the complications of pre-eclampsia?

Maternal and foetal.

A
  • Maternal:
    • Eclampsia
    • Cerebrovascular accident (CVA)
    • Haemolysis, elevated liver enzymes and low platelet count (HELLP)
    • Disseminated intravascular coagulation (DIC)
    • Liver failure
    • Renal failure
    • Pulmonary oedema
  • Foetal:
    • Intrauterine growth restriction (IUGR)
    • Pre-term birth
    • Placental abruption
    • Hypoxia
How well did you know this?
1
Not at all
2
3
4
5
Perfectly
34
Q

Define eclampsia.

A

the occurrence of one or more generalized convulsions and/or coma in the setting of preeclampsia and in the absence of other neurologic conditions

How well did you know this?
1
Not at all
2
3
4
5
Perfectly
35
Q

What are the clinical manifestations of eclampsia?

A
  • eclampsia is a clinical diagnosis
  • typically tonic-clonic and lasting 60-75 s
  • one of the signs of an impending seizure is hyperreflexia
  • symptoms that may occur before the seizure include persistent frontal or occipital headache, blurred vision, photophobia, right upper quadrant or epigastric pain, and altered mental status
  • in up to one third of cases, there is no proteinuria or blood pressure is <140/90 mmHg prior to the seizure
  • In general, women with typical eclamptic seizures who do not have focal neurologic deficits or prolonged coma do not require diagnostic evaluation including imaging
How well did you know this?
1
Not at all
2
3
4
5
Perfectly
36
Q

How is eclampsia managed?

A
  • ABCs
  • roll patient into LLDP
  • supplemental O2 via face mask to treat hypoxemia due to hypoventilation during convulsive episode
  • aggressive antihypertensive therapy for sustained diastolic pressures ≥105 mmHg or systolic blood pressures ≥160 mmHg with hydralazine or labetalol
  • prevention of recurrent convulsions: to prevent further seizures and the possible complications of repeated seizure activity (e.g. rhabdomyolysis, metabolic acidosis, aspiration pneumonitis, etc.)
  • MgSO4 is now the drug of choice, with previously used agents including diazepam and phenytoin
  • the definitive treatment of eclampsia is DELIVERY, irrespective of gestational age, to reduce the risk of maternal morbidity and mortality from complications of the disease
  • mode of delivery is dependent on clinical situation and fetal-maternal condition
How well did you know this?
1
Not at all
2
3
4
5
Perfectly
37
Q

What symptoms would you expect in a patient how has magnesium sulphate toxicity? and the Rx for toxicity

A
  • Flushing
  • Hyporeflexia
  • Somnolence
  • Respiratory and cardiac depression
  • Weakness
  • Note: Increased risk of toxicity with concurrent calcium channel blocker use or renal disease
  • Rx:
    • Stop MgSO4
    • Calcium gluconate 10% in 10 mL IV
How well did you know this?
1
Not at all
2
3
4
5
Perfectly
38
Q

Describe the prevalence of N/V in pregnancy?

A
  • affects 50-90% of pregnant women
  • often limited to T1 but may persist
How well did you know this?
1
Not at all
2
3
4
5
Perfectly
39
Q

What is the management for N/V in pregnancy?

A
  • rule out other causes of N/V
  • weigh frequently, assess level of hydration, test urine for ketones
  • non-pharmacological
    • Avoid mixing fluids and solids, frequent small meals
    • stop prenatal vitamins (folic acid must continue until >12 wk)
    • increase sleep/rest
    • ginger (maximum 1,000 mg/d)
    • acupuncture, acupressure
  • pharmacological
    • first line: Diclectin® (10 mg doxylamine succinate with vitamin B6) 4 tablets PO daily to maximum of 8 tablets
    • if no improvement, try dimenhydrinate (50-100 mg q4-6h PO), followed by hydroxyzine, pyridoxine, phenothiazine, or metoclopramide
    • vitamin B6 lollipops
    • if patient dehydrated, assess fluid replacement needs and resuscitate accordingly
  • severe/refractory:
    • consider homecare with IV fluids and parenteral anti-emetics, hospitalization
How well did you know this?
1
Not at all
2
3
4
5
Perfectly
40
Q

Define hyperemesis gravidarum.

A
  • intractable N/V, usually presents in T1 then diminishes; occasionally persists throughout pregnancy
  • affects ~1% of pregnancies
How well did you know this?
1
Not at all
2
3
4
5
Perfectly
41
Q

What are the DDx for excessive vomiting in pregnancy?

A
  • Infections: UTI, hepatitis, meningitis, gastroenteritis
  • GI: appendicitis, cholecystitis, pancreatitis, fatty liver, peptic ulcer, SBO
  • metabolic: thyrotoxicosis, Addison’s, DKA, hyperparathyroidism
  • drugs
  • gestational trophoblastic disease (molar pregnancy, choriocarcinoma)
How well did you know this?
1
Not at all
2
3
4
5
Perfectly
42
Q

What Ix need to be done in a patient wiht hyperemesis gravidarum?

A
  • rule out systemic causes: GI inflammation, pyelonephritis, thyrotoxicosis
  • rule out obstetrical causes: multiple gestation, GTN, HELLP syndrome
  • FBC, UEC LFTs
  • urinalysis
  • US
How well did you know this?
1
Not at all
2
3
4
5
Perfectly
43
Q

How is hyperemesis gravidarum managed?

A
  • thiamine supplementation may be indicated
  • non-pharmacological (see Nausea and Vomiting)
  • pharmacological options
    • Diclectin® (for dosage, see Nausea and Vomiting)
    • Dimenhydrinate can be safely used as an adjunct to Diclectin® (1 suppository bid or 25 mg PO qid)
    • other adjuncts: hydroxyzine, pyridoxine, phenothiazine, metoclopramide
    • also consider: ondansetron or methylprednisolone
    • if severe: admit to hospital, NPO initially then small frequent meals, correct hypovolemia, electrolyte disturbance, and ketosis, TPN (if very severe) to reverse catabolic state
How well did you know this?
1
Not at all
2
3
4
5
Perfectly
44
Q

What are the complications of hyperemesis gravidarum?

A
  • maternal
    • dehydration, electrolyte and acid-base disturbances
    • Mallory-Weiss tear
    • Wernicke’s encephalopathy, if protracted course
    • death
  • fetal: usually none, IUGR is 15x more common in women losing >5% of pre-pregnancy weight
How well did you know this?
1
Not at all
2
3
4
5
Perfectly
45
Q

Describe the pathogenesis of gestational diabetes mellitus.

A
  • usually around 24-28 wk GA
  • anti-insulin factors produced by placenta and high maternal cortisol levels create increased peripheral insulin resistance → higher fasting glucose
    → leading to GDM and/or exacerbating pre-existing DM
How well did you know this?
1
Not at all
2
3
4
5
Perfectly
46
Q

What is the management plan for ladies with T1DM and T2DM who are pregnant or planning on becoming pregnant?

A

Preconception

  • pre-plan and refer to high-risk clinic
  • optimize glycemic control
  • counsel patient re: potential risks and complications
  • evaluate for diabetic retinopathy, neuropathy, CAD

Pregnancy

  • if already on oral medication, generally switch to insulin therapy
    • continuing glyburide or metformin controversial
    • teratogenicity unknown for other oral anti-hyperglycemics
  • tight glycemic control
    • insulin dosage may need to be adjusted in type 2 due to increased demand and increased insulin resistance
  • monitor as for normal pregnancy plus initial 24 h urine protein and creatinine clearance, retinal exam, HbA1c
    • HbA1c: >140% of pre-pregnancy value associated with increased risk of spontaneous abortion and congenital malformations
  • increased fetal surveillance (BPP, NST), consider fetal ECHO to look for cardiac abnormalities

Labor

  • timing of delivery depends on fetal and maternal health and risk factors (i.e. must consider size of baby, lung maturity, maternal blood glucose, and blood pressure control)
  • can wait for spontaneous labor if blood glucose well-controlled and BPP normal
  • induce by 40 wk
  • type of delivery
    • increased risk of cephalopelvic disproportion (CPD) and shoulder dystocia with babies >4,000 g (8.8 lbs)
    • elective C/S for predicted birthweight >4,500 g (9.9 lbs) (controversial)
  • monitoring
    • during labor monitor blood glucose q1h with patient on insulin and dextrose drip
    • aim for blood glucose between 60-120 mg/dL to reduce the risk of neonatal hypoglycemia

Postpartum

  • insulin requirements dramatically drop with expulsion of placenta (source of insulin antagonists)
  • no insulin is required for 48-72 h postpartum in most type 1 DM
  • monitor glucose q6h, restart insulin at two-thirds of pre-pregnancy dosage when glucose >144 mg/dL
How well did you know this?
1
Not at all
2
3
4
5
Perfectly
47
Q

What are the risk factors for gestational diabetes?

A
  • Age >25 yr
  • Obesity
  • Ethnicity (Aboriginal, Hispanic, Asian, African)
  • FHx of DM
  • Previous history of GDM
  • Previous child with birthweight >4.0 kg
  • Polycystic ovarian syndrome
  • Current use of glucocorticoids
  • Essential HTN or pregnancy-related HTN
How well did you know this?
1
Not at all
2
3
4
5
Perfectly
48
Q

What is the management of a lady with gestational diabetes?

A
  • first line is management through diet modification and increased physical activity
  • initiate insulin therapy if glycemic targets not achieved within 2 wk of lifestyle modification alone. Common regime:
    • short acting insulin before meals PRN
    • medium acting insulin at bedtime if fasting BG are elevated
  • glycemic targets (PG = plasma glucose): FPG ≤5.0 mmol/L, 1h PG ≤7.4 mmol/LL, 2h PG ≤6.7 mmol/L
  • Oral agents: metformin and glibenclamide have not been approved for use in pregnancy in Australia
  • stop insulin and diabetic diet postpartum
  • follow-up with 75 g OGTT 6 -12 weeks postpartum
How well did you know this?
1
Not at all
2
3
4
5
Perfectly
49
Q

What are the complications of DM in pregnancy for the mother?

A
  • Obstetric
    • HTN/preeclampsia (especially if pre-existing nephropathy/proteinuria): insulin resistance is implicated in etiology of HTN
    • Polyhydramnios: maternal hyperglycemia leads to fetal hyperglycemia, which leads to fetal polyuria (a major source of amniotic fluid).
  • Diabetic Emergencies
    • Hypoglycemia
    • Ketoacidosis
    • Diabetic coma
  • End-Organ Involvement or Deterioration (occur in T1DM and T2DM, not in GDM)
    • Retinopathy
    • Nephropathy
  • Other
    • Pyelonephritis/UTI: glucosuria provides a culture medium for E. coli and other bacteria
    • Increased incidence of spontaneous abortion (in T1DM and T2DM, not in GDM): related to pre-conception glycemic control
How well did you know this?
1
Not at all
2
3
4
5
Perfectly
50
Q

What are the complications of DM in pregnancy for the foetus?

A
  • Growth Abnormalities
    • Macrosomia: maternal hyperglycemia leads to fetal hyperinsulinism resulting in accelerated anabolism
    • IUGR: due to placental vascular insufficiency
  • Delayed Organ Maturity
    • Fetal lung immaturity: hyperglycemia interferes with surfactant synthesis (respiratory distress syndrome)
  • Congenital Anomalies (occur in T1DM and T2DM, not in GDM)
    • 2-7x increased risk of congenital heart disease, neural tube defect, GU (cystic kidneys), GI (anal and duodenal atresia), and MSK (sacral agenesis) anomalies due to hyperglycemia
    • Note: Pregnancies complicated by GDM do not manifest an increased risk of congenital anomalies because GDM develops after the critical period of organogenesis (in T1)
  • Labor and Delivery
    • Preterm labor/prematurity: most commonly in patients with HTN/preeclampsia
    • Preterm labor is associated with poor glycemic control but the exact mechanism is unknown
    • Increased incidence of stillbirth
    • Birth trauma: due to macrosomia, can lead to difficult vaginal delivery and shoulder dystocia
How well did you know this?
1
Not at all
2
3
4
5
Perfectly
51
Q

What are the complications of DM in pregnancy for the neonate?

A
  • Hypoglycemia: due to pancreatic hyperplasia and excess insulin secretion in the neonate
  • Hyperbilirubinemia and jaundice: due to prematurity and polycythemia
  • Hypocalcemia: exact pathophysiology not understood, may be related to functional hypoparathyroidism
  • Polycythemia: hyperglycemia stimulates fetal erythropoietin production
How well did you know this?
1
Not at all
2
3
4
5
Perfectly
52
Q

What is the clinical presentation and significance of macrosomia?

A
  • Infant weight >90th percentile for a particular gestational age or >4,000g
  • Clinical presentation
    • Mother: obese, excessive weight gain during pregnancy, uterus large for dates, Leopold manoeuvres may give appreciation of foetal size
    • Foetus: as per definition
  • Significance
    • Mother: increased risk of birth canal lacerations, risk of caesarean delivery and associated risks
    • Foetus: cephalopelvic disproportion (baby’s head or body too large to fit through mother’s pelvis), birth trauma (shoulder dystocia, foetal bone fracture, brachial plexua injuries), increased risk of
      perinatal mortality
    • Neonate: hypoglycaemia, haematologic disturbances, electrolyte disturbances
How well did you know this?
1
Not at all
2
3
4
5
Perfectly
53
Q

What is the clinical presentation and significance of polyhydramnios?

A
  • Excess of amniotic fluid in the amniotic sac, typically diagnosed with the amniotic fluid index (AFI) >24cm
  • Clinical presentation
    • Mother: increased abdominal size out of proportion for weight gain and dates, uterus large for dates, shiny skin with striae (in severe), dyspnoea, chest heaviness
    • Foetus: difficulty palpating foetal parts and hearing foetal heart rate
  • Significance
    • Mother: pressure from over-distended uterus (dyspnoea, oedema, hydronephrosis)
    • Foetus: cord prolapse, placental abruption, mal-presentation, preterm labour, uterine dysfunction and post-partum haemorrhage, 2-5 fold increase in risk of perinatal mortality
How well did you know this?
1
Not at all
2
3
4
5
Perfectly
54
Q

Describe the epidemiology, Ix, Rx and implications of hypothyroidism in pregnancy?

A
  • Affects 1% of pregnant women, most due to Hashimoto’s thyroiditis or thyroid surgery in areas where iodine deficiency is not an issue
  • Ix: thyroid function test (TFT), in pregnancy it is normal for TSH to be below classic lower limit of normal, free T4 tends to increase in 1st trimester and then decrease later in pregnancy
  • Rx replacement with thyroxine, TSH monitored 6-weekly
  • Implications for foetus/neonate: untreated disease is rare as anovulation is usual but is associated with a high perinatal mortality; even subclinical hypothyroidism associated with miscarriage, preterm delivery and intellectual impairment in childhood
How well did you know this?
1
Not at all
2
3
4
5
Perfectly
55
Q

Describe the epidemiology, Ix, Rx and implications of hyperthyroidism in pregnancy?

A
  • Affects 0.4% of pregnant women, usually due to Grave’s disease and gestational thyrotoxicosis
  • Ix: thyroid function test (TFT)
  • Rx:
    • Propylthiouracil (PTU) treatment of choice in pre-natal period or 1st trimester, may be switched to carbimazole after this time, both safe in breastfeeding
    • Poorly controlled disease risks a thyroid storm, usually near or at delivery
  • Implications for foetus/neonate: symptoms of hyperthyroidism including congestive heart failure, inadequately treated disease increases peri-natal mortality, PTU can cause neonatal hypothyroidism
How well did you know this?
1
Not at all
2
3
4
5
Perfectly
56
Q

What are the risk factors for venous thrombosis in pregnancy?

A
  • Usual risk: previous VTE, age >35, obesity, infection, bedrest/immobility, shock/dehydration, thrombophilias.
  • Specific for pregnancy:
    • Hypercoagulability:
      • Increased factors: II, V, VII, VIII, IX, X, XII, fibrinogen
      • Increased platelet aggregation
      • Decreased protein S, tPA, factors XI, XIII
    • Stasis:
      • Increased resistance to activated protein C
      • Antithrombin can be normal or reduced
      • Increased venous distensibility
      • Decreased venous tone 50% decrease in venous flow in
        lower extremity by T3
      • Uterus is mechanical impediment to venous return
    • Endothelial:
      • Vascular damage at delivery (C/S or SVD)
      • Uterine instrumentation
      • Peripartum pelvic surgery
How well did you know this?
1
Not at all
2
3
4
5
Perfectly
57
Q

What is Virchow’s trid for VTE?

A
  • Hypercoagulable state
  • Stasis
  • Endothelial damage
How well did you know this?
1
Not at all
2
3
4
5
Perfectly
58
Q

What Ix need to be done in a lady with suspected VTE?

A
  • duplex venous Doppler sonography for DVT
  • CXR and V/Q scan or spiral CT for PE
How well did you know this?
1
Not at all
2
3
4
5
Perfectly
59
Q

What is the Management for VTE in pregnancy?

And a plan for future pregnancies?

A
  • before initiating treatment, obtain a baseline FBC and aPTT
  • warfarin is contraindicated during pregnancy
  • unfractionated heparin
    • bolus of 5,000 IU followed by an infusion of ~30,000 IU/24h
    • measure aPTT 6 h after the bolus
    • maintain aPTT at a therapeutic level (1.5-2x normal)
    • repeat q24h once therapeutic
    • heparin-induced thrombocytopenia (HIT) uncommon (3%) but serious complication
    • LMWH can also be used in pregnancy
  • compression stockings
  • poor evidence to support a recommendation for or against avoidance of prolonged sitting
  • VTE prophylaxis
    • women on long-term anticoagulation: full therapeutic anticoagulation throughout pregnancy and for 6-12 wk postpartum
    • women with a non-active PMHx of VTE: unfractionated heparin regimens suggested
  • routine VTE prophylaxis
    • insufficient evidence in pregnancy to recommend routine use of LMWH
    • current prophylaxis regimens for acquired thrombophilias (e.g. APS syndrome) include low dose Aspirin® in conjunction with prophylactic heparin
How well did you know this?
1
Not at all
2
3
4
5
Perfectly
60
Q

Define anaemia in pregnancy and its prevalence in pregnancy?

A
  • WHO defines anaemia as Hb <110g/L in pregnancy and Hb <100g/L postpartum; it is recognised that during the 2nd trimester of pregnancy, Hb concentrations diminish by approximately 5g/L
  • WHO estimates that 12% of pregnant women in Australia have anaemia, with iron deficiency the most common cause, and megaloblastic anaemia second more common (folate more common than B12)
How well did you know this?
1
Not at all
2
3
4
5
Perfectly
61
Q

Discuss Fe decificency anaemia in pregnancy.

Pregnancy demands, Ix, Rx.

A
  • Increased risk of pre-term delivery, low birth weight, stillbirth and newborn death
  • Pregnancy demands
    • Physiological iron requirements at 3 times high in pregnancy than they are in menstruating women, with increasing demand as pregnancy advances
    • ~600mg of elemental iron is required to the increase in red cell mass during pregnancy and a further 300mg for the foetus, RDI of iron for the latter half of pregnancy is 30mg
  • Routine administration of iron is not recommended due to lack of evidence of outcomes
  • Investigations
    • FBC should be assessed at booking and 28wk, and those anaemic should have serum ferritin checked and offered a trial of therapeutic iron
    • Hb can by normocytic and normochromic in early stages, otherwise microcytic and hypochromic
  • Treatment
    • All women should have counselling regarding diet in pregnancy including details of iron rich food sources and factors that may inhibit (tannins in tea/coffee, calcium) or promote iron
      absorption (vitamin C, haem iron)
    • In iron deficiency anaemia the oral dose should be 100-200mg of elemental iron daily, replacement should continue for 3m or until 6wk postpartum
How well did you know this?
1
Not at all
2
3
4
5
Perfectly
62
Q

Discuss B12 and folate decificency anaemia in pregnancy.

Pregnancy demands, Ix, Rx.

A
  • Pregnancy demands
    • Folate and its co-factor vitamin B12 are required for DNA synthesis and cell division
    • During pregnancy, requirements are increased approximately 5-10 fold and stores may be exhausted if increased folate intake does not occur
    • B12 deficiency may result in irreversible neurological damage to the breastfed infant
  • Vegetarians and vegans should be supplemented with vitamin B12 in pregnancy and lactation, the RDI is 6 µg/d
  • Investigations
    • FBC at booking and 28wk, will likely see megaloblastic anaemia (increased MCV) if deficiency, but if combined with iron deficiency may present as normocytic
    • Treatment
      • If folate deficiency supplemental folate given at 5mg per day and continued through pregnancy
      • If B12 deficiency consult a haematologist/physician for advice regarding IM vitamin B12 injections
      • Foods high in folic acid include lightly cooked or raw green vegetables and fish; foods high in B12 include shellfish, liver, fish, crab, fortified soy products and cereals, red meat, dairy, eggs
How well did you know this?
1
Not at all
2
3
4
5
Perfectly
63
Q

Discuss UTIs in pregnancy.

Prevalence, Management, and when Ix should be done.

A
  • Aymptomatic bacteriuria
    • Affects 5% of women, but in pregnancy is more likely (20%) to lead to pyelonephritis
    • Management
      • The urine should be cultured at the booking visit, and asymptomatic bacteriuria treated
      • Subsequently, culture is performed if nitrites are detected on routine analysis
    • Pyelonphritis – affects 1-2% of women, causing loin pain, rigors, vomiting and fever, and requires treatment with IV antibiotics
    • E. coli accounts for 75% and is often resistant to amoxicillin
How well did you know this?
1
Not at all
2
3
4
5
Perfectly
64
Q

Describe the pathogenesis of foetal hydrops and the causes?

A
  • Occurs when extra fluid accumulates in two or more areas of the foetus
  • Causes
    • Immune: due to anaemia and haemolysis as a result of antibodies including rhesus disease
    • Non-immune
      • Chromosomal abnormalities e.g. trisomy 21
      • Structural abnormalities e.g. pleural effusions
      • Cardiac abnormalities or arrhythmias
      • Anaemia causing cardiac failure e.g. parvovirus, haemorrhage or foetal α-thalassaemia major
      • Twin-twin transfusion syndrome in chorionic twins causes hydrops in severe cases
How well did you know this?
1
Not at all
2
3
4
5
Perfectly
65
Q

What Ix need to be done in foetal hydrops?

A
  • U/S assessment, including echocardiography and assessment of the middle cerebral artery
  • Maternal blood taken for Kleihauer test and parvovirus, CMV and toxoplasmosis IgM testing
  • Foetal blood sampling performed if anaemia is suspected
  • Foetal blood sampling or amniocentesis performed for karytotyping
How well did you know this?
1
Not at all
2
3
4
5
Perfectly
66
Q

What is the Rx for foetal hydrops?

A
  • Treatment or prognosis depends on the cause – cure only possible where anaemia (transfusion) or compression by fluid collection such as pleural effusions (vesicoamnitoic shunting) have caused hydrops
How well did you know this?
1
Not at all
2
3
4
5
Perfectly
67
Q

Define Rhesus isoimmunisation and the pathophysiology?

A
  • Definition – antibodies (Ab) produced against a specific RBC antigen (Ag) as a result of antigenic stimulation with RBC of another individual
  • Pathophysiology
    • Blood groups – blood classified according to its ABO and rhesus genotype, the immune system of those that are rhesus negative (and do not express D antigen) will recognise the D antigen as foreign if they are exposed to it
    • Sensitisation
      • Small amounts of foetal blood cross the placenta and enter maternal circulation during sensitising events, and if the foetus is rhesus positive and mother rhesus negative, the mother
        will mount an immune response (sensitisation) creating anti-D antibodies
      • Potentially sensitising events – incompatible blood transfusions, previous foetal-maternal transplacental haemorrhage (e.g. ectopic pregnancy), invasive procedures in pregnancy (e.g. pre-natal diagnosis, cerclage, D&C), any type of abortion, labour and delivery
    • Haemolysis
      • Immunity is permanent, and if the mother’s immune system is again exposed to the antigen (e.g. a subsequent pregnancy), large numbers of antibodies are rapidly created
      • These antibodies can cross the placenta and bind to foetal RBS, which are then destroyed in the foetal reticuloendothelial system
      • This can cause haemolytic anaemia and ultimately death (rhesus haemolytic disease)
How well did you know this?
1
Not at all
2
3
4
5
Perfectly
68
Q

How is rhesus isoimmunisation prevented?

A
  • Antenatal
    • Check all women for antibodies at booking and 28wk
    • Anti-D should be given to all women who are rhesus negative at 28wk, which will reduce the rate of isoimmunisation in a first pregnancy from 1.5% to 0.2%
    • Anti-D also given to such women within 72h (up to 10d) of any sensitising event
  • Postnatal
    • Neonate’s blood groups is checked and if rhesus positive, anti-D given to the mother within 72h of delivery
    • A Kelihauer test, to assess the number of foetal cells in the maternal circulation, is also performed within 2h or birth to detect occasional larger foeto-maternal haemorrhages that require larger doses of anti-D to ‘mop up’
How well did you know this?
1
Not at all
2
3
4
5
Perfectly
69
Q

What is the neonatal sequelae of Rhesus isoimmunisation?

A
  • Mild disease – neonatal jaundice only
  • Moderate – sufficiency haemolysis to cause neonatal anaemia (haemolytic disease of the newborn)
  • Severe – in utero anaemia, cardiac failure, ascites, oedema (hydrops) and foetal death
How well did you know this?
1
Not at all
2
3
4
5
Perfectly
70
Q

Define pre-term delivery and its complications.

A
  • Definition – delivery between 24 and 37 weeks’ gestation, risks greatest before 34wk
  • Complications
    • Neonatal – occupancy in NICU, perinatal mortality, cerebral palsy, chronic lung disease, blindness, minor disability, less developed immune system (note that preterm infants are more at risk for
      infection because of this, but also because many have interventions that make them vulnerable)
    • Maternal – caesarean section more common, if infection associated with pre-term labour can caused severe maternal illness
How well did you know this?
1
Not at all
2
3
4
5
Perfectly
71
Q

What are the causes of pre-term labour?

A
  • idiopathic (most common)
  • maternal:
    • infection (recurrent pyelonephritis, untreated bacteriuria, chorioamnionitis)
    • genital infection (bacterial vaginosis is associated with a 2-fold increase in relative risk of preterm birth)
    • HTN, DM, chronic illness
    • mechanical factors: previous obstetric, gynecological, and
      abdominal surgeries
    • socio-environmental (poor nutrition, smoking, drugs, alcohol, stress)
  • maternal-fetal:
    • PPROM (common)
    • polyhydramnios
    • placenta previa
    • placental abruption
    • placental insufficiency
  • fetal: multiple gestation, congenital abnormalities of fetus, fetal hydrops
  • uterine: incompetent cervix, excessive enlargement (hydramnios), malformations (leiomyomas, septate uterus)
72
Q

What Ix can be done for suspected pre-term labour?

A
  • Assess foetal state – cardiotocography (CTG) and ultrasound
  • Assess the likelihood of delivery
    • If cervix is uneffaced, foetal fibronectin assay – negative result means delivery unlikely
    • Transvaginal scanning (TVS) of cervical length – >15mm means delivery unlikely
  • Look for infection – vaginal swabs should be taken, CRP usually high, steroids will cause WCC to rise
73
Q

What is the management of a suspected pre-term labour?

What are requirements and contraindications for tocolytics.

A
  • Initial
    • transfer to appropriate facility if stable
    • hydration (NS at 150 mL/h)
    • bed rest in LLDP
    • sedation (morphine)
    • avoid repeated pelvic exams (increased infection risk)
    • U/S examination of fetus (GA, BPP, position, placenta location, estimated fetal weight)
    • prophylactic antibiotics; controversial but may help delay delivery, important to consider if PPROM
  • Suppression of Labor – Tocolysis
    • does not inhibit preterm labor completely, but may buy time (used for <48 h) to allow for betamethasone valerate (Celestone®) and/or transfer to appropriate center for care of the premature infant
    • requirements (all must be satisfied)
      • preterm labor
      • live, immature fetus, intact membranes, cervical dilatation of <4cm
      • absence of maternal or fetal contraindications
    • contraindications
      • maternal: bleeding (placenta previa or abruption), maternal disease (HTN, DM, heart disease), preeclampsia or eclampsia, chorioamnionitis
      • foetal: erythroblastosis fetalis, severe congenital anomalies, foetal distress/demise, IUGR, multiple gestation (relative)
  • Enhancement of Foetal Pulmonary Maturity
    • betamethasone or dexamethasone
    • 28-34 wk GA: reduces incidence of RDS
    • 24-28 wk GA: reduces severity of RDS, overall mortality, and rate of IVH
    • specific maternal contraindications: active TB
74
Q

Define PROM in all its forms.

A
  • PROM or amniorrhexis: rupture of membranes prior to labor at any GA
  • prolonged ROM: >24 h elapsed between rupture of membranes and onset of labor
  • preterm ROM: ROM occurring before 37 wk gestation (associated with PTL)
  • PPROM: rupture of membranes before 37 wk AND prior to onset of labor
75
Q

What are the causes of PPROM?

A

Same as pre-term labour

  • idiopathic (most common)
  • maternal: infection (recurrent pyelonephritis, untreated bacteriuria, chorioamnionitis), genital infection (bacterial vaginosis is associated with a 2-fold increase in relative risk of preterm birth), HTN, DM, chronic illness, mechanical factors, previous obstetric, gynecological, and
    abdominal surgeries, socio-environmental (poor nutrition, smoking, drugs, alcohol, stress)
  • maternal-fetal: PPROM (common), polyhydramnios, placenta previa, placental abruption, or
    placental insufficiency
  • fetal: multiple gestation, congenital abnormalities of fetus, fetal hydrops
  • uterine: incompetent cervix, excessive enlargement (hydramnios), malformations (leiomyomas, septate uterus)
76
Q

What Ix need to be done in PPROM?

A
  • U/S: can show reduced liquor, but volume can also be normal as foetal urine production continues
  • To look for infection: high vaginal swab (HVS), FBS and CRP taken; in doubtful cases amniocentesis with gram staining and culture occasionally used
  • Foetal well-being: assessed by CTG, persistent foetal tachycardia is suggestive of infection
77
Q

What is the management of PPROM?

A
  • admit for expectant management and monitor vitals q4h, daily BPP and WBC count
  • avoid introducing infection with examinations (do NOT do a bimanual exam)
  • cultures (cervix for GC, lower vagina for GBS)
  • assess fetal lung maturity by L/S ratio of amniotic fluid
    • consider administration of betamethasone valerate (Celestone®) to accelerate maturity if <32 wk and no evidence of infection
    • consider tocolysis for 48 h to permit administration of steroids if PPROM induces labor
  • if not in labor or labor not indicated, consider antibiotics (controversial)
    • studies show broad spectrum coverage increases the time to onset of labor from PROM by 5-7 d with no increase in maternal or neonatal morbidity or mortality
  • deliver urgently if evidence of foetal distress and/or chorioamnionitis
78
Q

What is the causative organism for chorioamnionitis?

A

GBS

79
Q

What ABx should be used for PPROM?

A
  • Without chorioamnionitis
    • amoxy/ampicillin 2 g IV, 6-hourly for 48 hours, followed by amoxycillin 250 mg orally, 8-hourly for a total of 7 days (IV + oral)
      • PLUS erythromycin 250 mg orally, 6-hourly for 7 days
  • With chorioamnionitis
    • Mild to moderate: amoxycillin+clavulanate 875+125 mg orally, 12-hourly for 14 days
      • PLUS doxycycline 100 mg orally, 12-hourly for 14 days.
    • Severe infection: amoxy/ampicillin 2 g IV, 6-hourly
      • PLUS gentamicin IV;
      • PLUS metronidazole 500 mg IV, 12-hourly.
80
Q

What is the prognosis for PROM?

A
  • varies with gestational age
  • 90% of women with PROM at 28-34 wk GA go into spontaneous labor within 1 wk
  • 50% of women with PROM at <26 wk GA go into spontaneous labor within 1 wk
  • complications: cord prolapse, intrauterine infection (chorioamnionitis), premature delivery, limb contracture
81
Q

Describe the anatomy of the foetal skull.

A
  • The head is oblong in transverse section
  • Its bones are not yet fused and, on vaginal examination, spaces between
    them are palpable as sutures and fontanelles
  • The anterior fontanelle (bregma) lies above the forehead, the posterior
    fontanelle (occiput) lies on the back of the top of the head
  • The head can be compressed in the pelvis because the sutures allow the
    bones to come together and even overlap slights (moulding)
82
Q

Describe the anatomy of the bony pelvis and soft tissues of the pelvis with respect to pregnancy.

A
  • The bony pelvis
    • Inlet – transverse diameter is about 13cm, wider than the 11cm antero-posterior (AP) diameter
    • Mid-cavity – almost round as the transverse and AP diameters are similar
    • Outlet – AP diameter (12.5cm) is greater than the transverse diameter (11cm)
    • Ischial spines – palpable vaginally in the lateral wall of the round mid-pelvis
  • The soft tissues
    • Cervical dilatation is a prerequisite for delivery and is dependent on
      contractions, the pressure of the foetal head on the cervix and the ability of the cervix to soften and allow distention
    • The soft tissues of the vagina and perineum need to be overcome in the second stage
85
Q

Define true and false labour?

A
  • true labor: regular, painful contractions of increasing intensity associated with progressive dilatation and effacement of cervix and descent of presenting part, or progression of station
    • preterm (>20 to <36+6 wk GA)
    • term (37-41+6 wk GA)
    • postterm (>42 wk GA)
  • false labor: Braxton-Hicks contractions
    • irregular contractions, with unchanged intensity and long intervals, occur throughout pregnancy and not associated with any dilatation, effacement, or descent
    • often relieved by rest or sedation
86
Q

Describe the first stage of labour.

A
  • latent phase
    • uterine contractions typically infrequent and irregular
    • slow cervical dilatation (usually to 3-4 cm) and effacement
  • active phase
    • rapid cervical dilatation to full dilatation (nulliparous ~1.2 cm/h, multiparous ~1.5 cm/h)
    • phase of maximum slope on cervical dilatation curve
    • painful, regular contractions q2-3min, lasting 45-60 s
    • contractions strongest at fundus, weakest at lower segment
87
Q

Describe the second stage of labour.

A
  • from full dilatation to delivery of the baby
  • mother feels a desire to bear down and push with each contraction
  • women may choose a comfortable position that enhances pushing efforts and delivery
    • upright (semi-sitting, squatting) and LLDP are supported in the literature
  • progress measured by descent
88
Q

Describe the third stage of labour.

A
  • separation and expulsion of the placenta
  • can last up to 30 min before intervention indicated
  • start oxytocin IV drip, or give 10 U IM or 5 mg IV push after delivery of anterior shoulder in anticipation of placental delivery, otherwise give after delivery of placenta
  • routine oxytocin administration in third stage of labor can reduce the risk of PPH by >40%
89
Q

Describe the fourth stage of labour.

A
  • first postpartum hour
  • monitor vital signs and bleeding
  • repair lacerations
  • ensure uterus is contracted (palpate uterus and monitor uterine bleeding)
  • inspect placenta for completeness and umbilical cord for presence of 2 arteries and 1 vein
  • 3rd and 4th stages of labor most dangerous to the mother (i.e. hemorrhage)
90
Q

Describe the options for pain relief in labour.

A
  • pain or anxiety leads to high endogenous catecholamines, which produce a direct inhibitory effect on uterine contractility

_​_Non-Pharmacologic Pain Relief Techniques

  • reduction of painful stimuli
    • maternal movement, position change, counter-pressure, abdominal compression
  • activation of peripheral sensory receptors
    • superficial heat and cold
    • immersion in water during labor
    • touch and massage, acupuncture, and acupressure
    • TENS
    • intradermal injection of sterile water
      • aromatherapy
  • enhancement of descending inhibitory pathways
    • attention focusing and distraction
    • hypnosis
    • music and audio analgesia
    • biofeedback
  • Pharmacologic Methods (see Anesthesia, A24)
    • nitrous oxide (e.g. self-administered Entonox®)
    • narcotics (usually combined with anti-emetic)
    • pudendal nerve block
    • perineal infiltration with local anesthetic
    • regional anesthesia (epidural block, combined spinal-epidural, spinal)
91
Q

Describe the partogram and its use in labour.

A
  • A partogram or partograph is a composite graphical record of key data (maternal and fetal) during labour entered against time on a single sheet of paper. Relevant measurements might include statistics such as cervical dilation, fetal heart rate, duration of labour and vital signs.
  • It is intended to provide an accurate record of the progress in labour, so that any delay or deviation from normal may be detected quickly and treated accordingly. However, a Cochrane review came to the conclusion that there is insufficient evidence to recommend partograms in standard labour management and care.
92
Q

50% of Downs syndrome births have congenital heart disease.

List in decreasing frequency the defects.

A
  1. Atrioventricular septal defect
  2. Ventricular septal defect
  3. Patent ductus arteriosus
  4. Tetralogy of Fallot, and
  5. Atrial septal defect.
93
Q

What are some common causes of failure to progress in labour (dystocia)?

A
  • Power (leading cause): contractions (hypotonic, incoordinate), inadequate maternal explusive
    efforts
  • Passenger: fetal position, attitude, size, anomalies (hydrocephalus)
  • Passage: pelvic structure (CPD), maternal soft tissue factors (tumors, full bladder or rectum, vaginal septum)
  • Psyche: hormones released in response to stress may contribute to dystocia; psychological and physiological stress should be evaluated as part of the management once dystocia has been diagnosed
94
Q

What methods can be used to induce labour?

A
  • Amniotomy
    • artificial rupture of membranes (amniotomy) to stimulate prostaglandin synthesis and secretion; may try this as initial measure if cervix is dilated
    • amniotomy plus intravenous oxytocin: more women delivered vaginally at 24 h than amniotomy alone (relative risk = 0.03) and had fewer instrumental vaginal deliveries (relative risk = 5.5)
  • Oxytocin
    • oxytocin: 10 U in 1L NS, run at 0.5-2 mU/min IV increasing by 1-2 mU/min q20-60min to a max of 36-48 mU/min
      • reduces rate of unsuccessful vaginal deliveries within 24 h when used alone (8.3% vs. 54%, RR 0.16)
      • ideal dosing regime of oxytocin is not known
      • current recommendations: use the minimum dose to achieve active labor and increase q30min as needed
      • reassessment should occur once a dose of 20 mU/min is reached
    • potential complications
      • hyperstimulation/tetanic contraction (may cause fetal distress or rupture of uterus)
      • ƒ uterine muscle fatigue, uterine atony (may result in PPH)
      • vasopressin-like action causing anti-diuresis
95
Q

Define what is meant by cervical ripening?

A
  • use of medications or other means to soften, efface, and dilate the cervix to increase likelihood of induction success
  • ripening of an unfavorable cervix (Bishop score <6) is warranted prior to induction of labor
96
Q

What methods can be used in cervical ripening?

A
  • intravaginal prostaglandin PGE2 gel: long and closed cervix
    • recommended dosing interval of prostaglandin gel is every 6 to 12 h up to 3 doses
  • intravaginal PGE2: long and closed cervix, may use if ROM
    • continuous release, can be removed if needed
    • controlled release PGE2
  • Foley catheter placement to mechanically dilate the cervix
97
Q

What are the implications to the foetus in dysfunctional labour?

A
  • Foetal distress and hypoxia
  • Foetal infection
  • Meconium aspiration
  • Foetal trauma
  • Foetal blood loss
98
Q

Describe what a partogram is and how it is used?

A
  • Used to record progress in dilatation of the cervix (± descent of the head) as assessed on vaginal examination, and is plotted against time
  • The usual minimum rate of dilatation is 1cm/h – ‘alert’ and ‘action’ lines on the partogram indicate slow progress, so aids identification abnormal progress
  • Records each hour – foetal heart rate, liquor colour, cervical dilatation, descent of the head, BP, pulse, temperature, urine (protein, acetone, volume), oxytocin and any other drugs/fluid given
99
Q

What are the non-pharmacologic techniques used to reduce pain in labour and birth?

A
  • Reduction of painful stimuli: maternal movement, position change, counter-pressure, abdominal compression
  • Activation of peripheral sensory receptors: Superficial heat and cold, immersion in water during labour, touch and massage, acupuncture and acupressure, TENS, intradermal injection of sterile water, aromatherapy
  • Enhancement of descending inhibitory pathways: Attention focussing and distraction, hypnosis, music and audio analgesia, biofeedback
100
Q

What are the pharmacological analgesics used in labour and birth?

A
  • Nitrous oxide (e.g. self-administered Entonox)
  • Narcotics – usually combined with anti-emetic
  • Pudendal nerve block
  • Perineal infiltration with local anaesthetic
  • Regional anaesthesia – epidural block, CSE, spinal
101
Q

What is the active management of labour?

A
  • Aim – to shorten the length of labour
  • Principles
    • Early diagnosis of labour2-ho
    • urly vaginal examinations
    • Augmentation – Early correction of slow progress
      • Artificially rupturing the membranes (ARM or amniotomy)
      • If this fails to further cervical dilatation in 1-2h, artificial oxytocin is administered IV and will usually increase cervical dilatation within 4h
    • Caesarean section by 12h if delivery is not imminent
102
Q

Define pre-term, term, port-term.

A
  • Pre-term – 20wk to 36+6wk gestation
  • Term – 37wk to 41+6wk gestation
  • Post-term (also known as post-dates, post-mature, overdue) – more than 42wk gestation
    • Risks of reduced placental perfusion, oligohydramnios, meconium aspiration
103
Q

What is Naegles’ rule?

A
  • To estimate the expected date of delivery (EDD), subtract 3m from the date of last menstrual period (LMP), add 7 days and 1 year
  • If a cycle is more or less than 28d, the EDD will be later and needs to be adjusted
104
Q

What are the indications for induction of labour?

A
  • Foetal: high-risk situations such as prolonged pregnancy, suspected IUGR or compromise, antepartum haemorrhage, poor obstetric history and pre-labour term rupture of the membranes
  • Materno-foetal indications: pre-eclampsia and maternal disease such as diabetes
  • Maternal indications: social reasons and in utero death
105
Q

What are the indications for operative vaginal delivery?

A
  • fetal
    • atypical or abnormal fetal heart rate tracing
    • consider if second stage is prolonged as this may be due to poor contractions or failure of fetal head to rotate
  • maternal
    • need to avoid voluntary expulsive effort (e.g. cardiac/cerebrovascular disease)
    • exhaustion, lack of cooperation, and excessive analgesia may impair pushing effort
106
Q

What are the risks and complcations of forceps and vacuum extraction?

A
  • Forceps:
    • Maternal: anaesthesia risk, lacerations, injury to bladder, uterus, or bone, pelvic nerve damage, PPH, infections
    • Fetal: fractures, facial nerve palsy, trauma to face/scalp, intracerebral haemorrhage, cephalohaematoma, cord compression
  • Vacuum:
    • Increased incidence of cephalohaematoma and retinal haemorrhages compared to forceps
    • Subgaleal hemorrhage, subaponeurotic haemorrhage, soft tissue trauma
107
Q

What are the indications for cesarean section?

A
  • maternal: obstruction, active herpetic lesion on vulva, invasive cervical cancer, previous uterine surgery, underlying maternal illness (eclampsia, HELLP syndrome, heart disease)
  • maternal-fetal: failure to progress, placental abruption or previa, vasa previa
  • faetal: abnormal faetal heart tracing, malpresentation, cord prolapse, certain congenital anomalies
108
Q

What are the types of incisions used in cesareans?

A
  • skin
    • transverse (i.e. Pfannensteil)
      • decreased exposure and slower entryŠ
      • improved strength and cosmesis
    • vertical midline
      • rapid peritoneal entry and increased exposure
      • Increased dehiscence
  • uterine
    • low transverse (most common): in noncontractile segment
      • decreased chance for rupture in subsequent pregnancies
    • low vertical
      • used for very preterm infants, poorly developed maternal lower uterine segment
    • classical (rare): in thick, contractile segment
      • used for transverse lie, faetal anomaly, >2 fetuses, lower segment adhesions, obstructing fibroid, morbidly obese patients
109
Q

What are the risks/complications of cesarean section?

A
  • anesthesia
  • hemorrhage (average blood loss ~1,000 cc)
  • infection (UTI, wound, endometritis)
    • single dose prophylactic antibiotic should be used (e.g. cefazolin 1-2g)
  • injury to surrounding structures (bowel, bladder, ureter, uterus)
  • thromboembolism
  • increased recovery time/hospital stay
  • maternal mortality (<0.1%)
110
Q

What is the name of the ligament that is obliterated in C section?

A

Urachus

111
Q

What are the layers of the rectus sheath above and below the arcuate line?

A
  • Above the arcuate line: external oblique, internal oblique, rectus abdominis, internal oblique, transversus abdominis
  • Below the arcuate line: external oblique, internal oblique, transversus abdominis, rectus abdominis
112
Q

Describe what a VBAC is and the contraindications to one?

A
  • Vaginal birth after cesarean (VBAC) - trial of labour after cesarean.
    • recommended after previous low transverse incision
    • success rate varies with indication for previous C/S (generally 60-80%)
    • risk of uterine rupture (<1% with low transverse incision)
  • Contraindications
    • previous classical, inverted T, or unknown uterine incision, or complete transection of uterus (6% risk of rupture)
    • history of hysterotomy or previous uterine rupture
    • multiple gestation
    • non-vertex presentation or placenta previa
    • inadequate facilities or personnel for emergency C/S
113
Q

Describe the epidemiology of mutliple gestations?

A
  • incidence of twins is 1/80 and triplets 1/6,400 in North America
  • 2/3 of twins are dizygotic (fraternal)
    • risk factors for dizygotic twins: IVF, increased maternal age, newly discontinued OCP, ethnicity (e.g. certain African regions)
  • monozygous twinning occurs at a constant rate worldwide (1/250)
  • determine zygosity by number of placentas, thickness of membranes, sex, blood type
114
Q

What are the complications associated with multiple gestations?

A
  • Maternal:
    • Hyperemesis gravidarum
    • GDM
    • Gestational HTN
    • Anemia
    • Increased physiological stress on all
    • systems
    • Increased compressive symptoms
    • C/S
  • Uteroplacental:
    • Increased PROM/PTL
    • Polyhydramnios
    • Placenta previa
    • Placental abruption
    • PPH (uterine atony)
    • Umbilical cord prolapse
    • Cord anomalies (velamentous insertion, 2 vessel cord)
  • Faetal:
    • Prematurity*
    • IUGR
    • Malpresentation
    • Congenital anomalies
    • Twin-twin transfusion
    • Increased perinatal morbidity and mortality
    • Twin interlocking
    • (twin A breech, twin B vertex)
    • Single fetal demise
115
Q

How are multiple gestations managed?

A
  • U/S determination of chorionicity must be done within first trimester (ideally 8-12 wk GA)
  • increased antenatal surveillance
    • serial U/S q2-3wk from 28 wk GA to assess growth (uncomplicated diamniotic dichorionitic)
    • increased frequency of ultrasounds in monochorionic diamniotic and monochorionic monoamniotic twins
    • Doppler flow studies weekly if discordant fetal growth (>30%)
      ƒ BPP as needed
  • may attempt vaginal delivery if twin A presents as vertex, otherwise C/S (40-50% of all twin deliveries, 15% of cases have twin A delivered vaginally and twin B delivered by C/S)
  • mode of delivery depends on fetal weight, GA, presentation
116
Q

What are the different types of twins and there variations in placentas?

A
  • Monozygotic twins – originate from the fertilisation and subsequent division of one egg
    • Dichorionic and diamniotic (DCDA) – one chorion and one placenta (amnion)
      • Fertilised egg splits in the first 3 days after fertilisation
      • Risk if pregnancy extends past 38-39wk
    • Monochorionic diamniotic (MCDA) – split occurs at days between days 4 and 8
      • Considered high-risk due to 3-5 fold increase in perinatal morbidity and mortality compared to DCDA pregnancies, risk of sudden unexpected stillbirth between 5-10% after 32wk
      • Twin to twin transfusion syndrome (TTTS) complicates about 15%, with discordant IUGR complicating an additional 25%
    • Monochorionic monoamniotic (MCMA) – split occurs after the 8th day post-fertilisation
      • At high risk of foetal death and neonatal morbidity, secondary to umbilical cord entanglement, prematurity, and congenital anomalies, risk of sudden stillbirth as with MCDA
    • Conjoined twins – division 13 days or later (extremely rare)
  • Dizygotic twins – originate from the fertilisation and development of two eggs, are always DCDA
117
Q

Define placenta praevia?

A

Abnormal location of the placenta near, partially, or completely over the internal cervical os

118
Q

What are the causes of vaginal bleeding in pregnancy?

A
  • bloody show (shedding of cervical mucous plug) – most common etiology in T3
  • placenta previa
  • abruptio placentae – most common pathological etiology in T3
  • vasa previa
  • cervical lesion (cervicitis, polyp, ectropion, cervical cancer)
  • uterine rupture
  • other: bleeding from bowel or bladder, placenta accreta, abnormal coagulation
119
Q

What are the key questions to ask in antepartum haemorrhage?

A
  • How much bleeding?
  • Are there contractions/cramping/pain?
  • Description? Colour, clotting, etc.
120
Q

Define placental abruption

A

Premature separation of a normally implanted placenta after 20 wk GA

121
Q

What are the clinical features of placetna previa?

A
  • PAINLESS bright red vaginal bleeding (recurrent), may be minimized and cease spontaneously, but can become catastrophic
  • mean onset of bleeding is 30 wk GA, but onset depends on degree of previa
  • physical exam
    • uterus soft and non-tender
    • presenting fetal part high or displaced
    • FHR usually normal
    • shock/anemia correspond to degree of apparent blood loss
  • complications
    • foetal
    • perinatal mortality low but still higher than with a normal pregnancy
    • prematurity (bleeding often dictates early C/S)
    • intrauterine hypoxia (acute or IUGR)
    • foetal malpresentation
    • PPROM
    • risk of fetal blood loss from placenta, especially if incised during C/S
  • maternal
    • <1% maternal mortality
    • haemorrhage and hypovolemic shock, anemia, acute renal failure, pituitary necrosis (Sheehan syndrome)
    • placenta accreta – especially if previous uterine surgery, anterior placenta previa
    • hysterectomy
122
Q

What Ix need to be done for placenta previa?

A
  • transvaginal U/S is more accurate than transabdominal U/S at diagnosing placenta previa at any gestational age
  • if the placenta lies between 20 mm of overlap and 20 mm away from the internal os after 26 wk regular transvaginal ultrasounds should be repeated at regular intervals – continued change in the placental location is likely
123
Q

How is placenta previa managed?

A
  • goal: keep pregnancy intrauterine until the risk of delivery < risk of continuing pregnancy
  • stabilize and monitor
    • maternal stabilisation: large bore IV with hydration, O2 for hypotensive patients
    • maternal monitoring: vitals, urine output, blood loss, blood work (haematocrit, CBC, INR/ PTT, platelets, fibrinogen, FDP, type and cross match)
    • electronic fetal monitoring
    • U/S assessment: when fetal and maternal condition permit, determine fetal viability, gestational age, and placental status/position
  • Rhogam® if mother is Rh negative
    • Kleihauer-Betke test to determine extent of fetomaternal transfusion so that appropriate dose of Rhogam® can be given
  • GA <37 wk and minimal bleeding: expectant management
    • admit to hospital
    • limited physical activity, no douches, enemas, or sexual intercourse
    • consider corticosteroids for fetal lung maturity
    • delivery when fetus is mature or hemorrhage dictates
  • GA ≥37 wk, profuse bleeding, or L/S ratio is >2:1 – deliver by C/S
124
Q

What are the classifications for placental abruption?

A
  • total (fetal death inevitable) vs. partial
  • external/revealed/apparent: blood dissects downward toward cervix
  • internal/concealed (20%): blood dissects upward toward fetus
  • most are mixed
125
Q

What is the most common cause of DIC in pregnancy?

A

Placental abruption

126
Q

How does placental abruption present?

A
  • PAINFUL (80%) vaginal bleeding (bleeding not always present if abruption is concealed), uterine tenderness, uterine contractions
  • pain: sudden onset, constant, localized to lower back and uterus
  • shock/anemia out of proportion to apparent blood loss
  • ± fetal distress, fetal demise (15% present with demise), bloody amniotic fluid (fetal presentation typically normal)
  • ± coagulopathy
127
Q

What are the complications of placental abruption?

A
  • foetal complications: perinatal mortality 25-60%, prematurity, intrauterine hypoxia
  • maternal complications: <1% maternal mortality, DIC (in 20% of abruptions), acute renal failure, anaemia, haemorrhagic shock, pituitary necrosis (Sheehan syndrome), amniotic fluid embolus
128
Q

What Ix need to be done for placental abruption?

A

clinical diagnosis, U/S not sensitive for diagnosing abruption (sensitivity = 15%)

129
Q

What is the management of placental abruption?

A
  • maternal stabilization: large bore IV with hydration, O2 for hypotensive patients
  • maternal monitoring: vitals, urine output, blood loss, blood work (haematocrit, FBC, PTT/PT, platelets, fibrinogen, FDP, group and hold)
  • Electronic Foetal Monitoring
  • blood products on hand (red cells, platelets, cryoprecipitate) because of DIC risk
  • Rhogam® if Rh negative
    • Kleihauer-Betke test may confirm abruption
  • mild abruption
    • GA <37 wk: use serial Hct to assess concealed bleeding, deliver when foetus is mature or when haemorrhage dictates
    • GA ≥37 wk: stabilize and deliver
  • moderate to severe abruption
    • hydrate and restore blood loss and correct coagulation defect if present
    • vaginal delivery if no contraindication and no evidence of foetal or maternal distress OR foetal demise
    • C/S if live foetus and foetal or maternal distress develops with fluid/blood replacement, labor fails to progress or if vaginal delivery otherwise contraindicated
130
Q

Define primary and secondary post portum haemorrhage?

A
  • Primary: loss of 500mL of blood after vaginal delivery or 1,000mL after Caesarean within first 24h.
  • Secondary: excessive blood loss from the genital tract occurring more than 24 hours to 6 weeks after delivery.
131
Q

What are the risk factors for post-partum haemorrhage?

And how are these risk factor mitigated?

A
  • Risk factors:
    • antepartum bleeding
    • use of Syntocinon (Oxytocin) in labour
    • prolonged labour, instrumental delivery
    • uterine abnormalities e.g. fibroids, placenta praevia, previous PPH, sepsis
  • Mitigation:
    • Antepartum checks on Hb (and Fe supplementation if required)
    • IV line
    • group and hold
    • fluid management during labour
    • active management of third stage
    • fundal massage
    • check placenta for completeness
    • check perineum and vagina for trauma
132
Q

What are the causes of post-partum haemorrhage?

HINT: 4 T’s

A
  • Tone
    • uterine atony
      • most common cause of PPH
      • avoid by giving oxytocin with delivery of the anterior shoulder or placenta
      • occurs within first 24 h
    • due to
      • labor (prolonged, precipitous, induced, augmented)
      • uterus (infection, over-distention)
      • placenta (abruption, previa)
      • maternal factors (grand multiparity, gestational HTN)
      • halothane anesthesia
  • Tissue
    • retained placental products
    • retained blood clots in an atonic uterus
    • gestational trophoblastic neoplasia
  • Trauma
    • laceration (vagina, cervix, uterus), episiotomy, hematoma (vaginal, vulvar, retroperitoneal), uterine rupture, uterine inversion
  • Thrombin
    • coagulopathy
      • most identified prior to delivery (low platelets increases risk)
      • includes hemophilia, DIC, Aspirin® use, ITP, TTP, vWD (most common)
      • therapeutic anti-coagulation
133
Q

What Ix need to be done on a pt with post-partum haemorrhage?

A
  • assess degree of blood loss and shock by clinical exam
  • explore uterus and lower genital tract for evidence of tone, tissue, or trauma
  • Bloods:
    • FBC, coags, INR/APTT, group and hold
134
Q

What Initial steps need to be taken with a pt with post-partum haemorrhage?

A
  • ABCs
  • 2 large bore IVs and crystalloids
  • FBC, coagulation profile, cross and type 4 units pRBCs
  • treat underlying cause
135
Q

What is the medical therapy for post-partum haemorrhage?

A
  • Tone:
    • oxytocin
    • ergotamine
    • misoprostol (side effect: pyrexia)
    • Prostol
  • Thrombin:
    • Tranexamic acid - antifibrinolytic
    • Replacement of platelets and clotting factors
136
Q

How can post-partum haemorrhage be locally controlled?

A
  • bimanual compression: elevate the uterus and massage through patient’s abdomen
  • uterine packing (mesh with antibiotic treatment)
  • Bakri Balloon for tamponade: may slow hemorrhage enough to allow time for correction of coagulopathy or for preparation of an OR
137
Q

What are the surgical options for post-partum haemorrhage?

A
  • Used for intractable PPH
    • D&C (beware of vigorous scraping which can lead to Asherman’s syndrome)
    • embolisation of uterine artery or internal iliac artery by interventional radiologist
    • laparotomy with bilateral ligation of uterine artery (may be effective), internal iliac artery (not proven), ovarian artery, or hypogastric artery
    • hysterectomy last option with angiographic embolisation if post-hysterectomy bleeding
138
Q

What are the common malpresentations?

A
  • Malpresentation – any presentation other than a vertex presentation (top of the head first)
    • Cephalic presentation (head first) – sinciput (forehead), brow (eyebrows), face, chin
    • Breech (buttocks or feet first) – complete breech, footling breech, frank breech
    • Shoulder presentation – arm, shoulder, trunk
139
Q

How is breech presentation diagnosed?

A
  • Diagnosis – only important from 37wk or if the patient is in labour
    • History – upper abdominal discomfort
    • Exam – head normally palpable and ballotable at the fundus
    • Investigation – U/S confirms the diagnosis, helps detection of other abnormalities and ensures prerequisites for external cephalic version (ECV) are met.
140
Q

How is breech presentation managed?

A
  • External cephalic version (ECV)
    • From 37wk, an attempt made to turn the baby to a cephalic presentation, success rate is about 50% and about 3% will turn back
    • Technique – under U/S guidance (and using tocometry if required), with both hands on the pelvis, the breech is disengaged from the pelvis, pushed upwards and to the side, and rotation in the form of a forward somersault is attempted; CTG performed straight after
    • Contraindication – not performed if the foetus is compromised, if vaginal delivery is contradicted anyway, multiple pregnancy, ruptured membranes, recent antepartum haemorrhage
  • Caesarean section
    • If ECV is failed or in contraindicated, or if breech presentation was missed
    • Some women still wish to deliver vaginally, breech presentation is often diagnosed in late labour only, and second twins often present as breech, and in these circumstances vaginal breech delivery is still appropriate
  • Vaginal breech birth
    • Intrapartum – pushing not encouraged until the buttocks visible, CTG advised, epidural common
    • In about 30% there is slow cervical dilatation – in these cases perform Caesarean section
    • Breech delivery
      • The foetus delivers with maternal effort as far as the umbilicus
      • The legs can be flexed out of the vagina, whilst the back is kept anterior, and once the scapula is visible the arms are hooked down by a finger over the shoulder
      • Mauriceau-Smellie-Veit manoeuvre – Once the back of the head is visible, the operator supports the entire weight of the foetus on one palm, with their finger in its mouth to guide
        the head over the perineum and maintain flexion, and the other hand against the occiput
141
Q

What are the risk factors for shoulder dystocia?

A
  • maternal: obesity, DM, multiparity
  • foetal: prolonged gestation, macrosomia
  • labour
    • prolonged 2nd stage
    • instrumental midpelvic delivery
142
Q

Define shoulder dystocia?

A
  • impaction of anterior shoulder of foetus against symphysis pubis after foetal head has been delivered
  • life threatening emergency
143
Q

What are the complications of shoulder dystocia?

A
  • Maternal: perineal injury, may result in PPH
  • Foetal: brachial plexus injury (e.g. Erb’s palsy which is permanent in 50%), fracture (clavicle, humerus, cervical spine), hypoxia (can be lethal)
144
Q

What are the clinical features of shoulder dystocia?

A
  • “turtle sign”: head delivered but retracts against inferior portion of pubic symphysis
145
Q

How is shoulder dystocia managed?

A
  • goal: to displace anterior shoulder from behind symphysis pubis; follow a stepwise approach of maneuvers until goal achieved
    • Position the mother’s legs in full flexion, hyperextended onto the abdomen (McRobert’s manoeuvre)
      • Applying suprapubic pressure in addition to McRobert’s manoeuvre works in 90% of cases
    • If the above fails, internal manoeuvres are required, necessitating episiotomy
      • Pressure behind the anterior shoulder, which can be combined with pressure on the anterior part of the posterior shoulder (Wood’s screw manoeuvre)
      • If this fails the posterior arm is grasped and the hand is brought down, the trunk should follow
  • other options
    • cleidotomy (deliberate fracture of neonatal clavicle)
    • Zavanelli maneuver: replacement of foetus into uterine cavity and emergent C/S
    • symphysiotomy
146
Q

What does the women need to be counselled about for future pregnancies?

A

Around 1 in 10 women will have shoulder dystocia again in the future

147
Q

What are the aetiologies of cord prolapse?

A
  • prematurity/PROM
  • foetal malpresentation (~50% of cases)
  • low-lying placenta
  • polyhydramnios
  • multiple gestation
  • cephalopelvic disproportion (CPD)
148
Q

What are the clinical features of cord prolapse?

A
  • Visible or palpable cord
  • Foetal heart rate changes – variable decelerations, bradycardia or both
149
Q

What are the effects on the foetus if the cord prolapses?

A

Cause cord compression between presenting part and pelvis, baby will become rapidly hypoxic

150
Q

What are the management principles of a cord prolapse?

A
  • O2 to mother, monitor foetal heart
  • Presenting part prevented from compressing the cord – pushed into the vagina by a digit, or tocolytics such as terbutaline are given
  • Keep cord warm and moist by replacing it into the vagina ± applying warm saline soaks
  • Position mother on all fours while preparation for delivery undertaken
  • Mode of delivery: Emergency Caesarean usually, instrumental delivery if the cervix fully dilated and the head is low
151
Q

What are the causes of secondary post-partum haemorrhage?

A
  • Retained products of conception
  • Infection (often secondary to retained products)
  • Lacerations, including episiotomy
  • Others (rare): Blood dyscrasias, Trophoblastic disease, Carcinoma of the cervix, Submucous fibroids (causing subinvolution), Placental site subinvolution
152
Q

How does secondary post-partum haemorrhage present?

A
  • can cause profound vasovagal response with vasodilation and hypovolaemic shock
  • may be disproportionate to maternal blood loss
153
Q

How is secondary post partum haemorrhage managed?

A
  • Bed rest and intravenous antibiotic therapy are the mainstays of treatment.
  • Curettage is not performed routinely (risk of uterine perforation or Asherman’s Syndrome). Evidence of retained products is suggested by subinvolution of the uterus, an open cervical os or a poor response to conservative management.
  • Oxytocics (eg: oral Ergometrine) have almost no part in the management.
  • If vaginal bleeding continues following curettage for secondary postpartum haemorrhage, then consider the need for a pelvic trans-vaginal ultrasound scan
154
Q

What are the normal physiological changes that occur in the puerperium?

Genital tract.

A
  • Immediately the placenta has separated, the uterus contracts and the fibres of the myometrium occlude the blood vessels that formerly supplied the placenta
  • Uterine size reduces over 6wk, within 10d the uterus is no longer palpable abdominally
  • The internal os of the cervix is closed by 3d
  • Lochia (discharge from the uterus) may be blood-stained for 4wk, but is thereafter yellow or white
155
Q

What are the normal physiological changes that occur in the puerperium?

Cardiovascular system.

A
  • Cardiac output and plasma volume decrease to prepregnant levels within a week
  • BP will return back to normal (if increased) and oedema will reduce within 6wk
156
Q

What are the normal physiological changes that occur in the puerperium?

Breast and lacatation

A
  • Suckling is the major stimulus for milk letdown, but the reflex can be conditioned
  • Impulse recognition (e.g. cry or sight of an infant) results in episodic oxytocin release from the posterior pituitary, resulting in milk ejection
  • Suckling reflects also affects the activity of the GnRH pulse generator, inhibiting gonadotropin release and thus ovulation does not typically occur; menstruation usually delayed by lactation (to ~3-6m), but occurs at about 6w if not lactating
157
Q

What are the common puerperal problems?

A
  • Breastfeeding problems
    • Inadequate milk, breast engorgement, nipple pain, mastitis, inverted nipples, maternal medications
  • Bladder dysfunction
    • Pelvic floor prolapse can occur after vaginal delivery
    • Stress or urge urinary incontinence common
    • Increased risk with instrumental delivery or prolonged second stage
  • Puerperal pain
    • “After pains” common in first 3 d due to uterine contractions
158
Q

What needs to be assessed at discharge post birth?

A
  • General – Mother and baby should not be separated, early mobilisation is encouraged
  • Daily monitoring – uterine involution and the lochia, BP, temperature, pulse, any perineal wound, careful fluid balance check should prevent retention if a woman has had an epidural
  • Investigations – FBC, iron prescribed if appropriate, usually in conjunction with laxatives
  • Analgesia – may be required for perineal pain (pelvic floor exercises can also help)
  • Midwife/doctor – whoever attended the delivery should visit the patient and discus the delivery (particularly if there has been an intervention) and the woman can ask questions about the labour
  • Discharge
    • Dependent on the mother’s wishes, the child may need to stay longer
    • Complete the blue book and give to woman with explanation, advise when baby vaccinations due
    • Advice on contraception given prior
    • GP should be alerted of any complications, and a postnatal plan for follow up for any psychiatric issues of pregnancy should be drawn up
159
Q

What are the advantages of breastfeeding?

A
  • Enhancement of mother-child bonding
  • Protection of baby against common health problems – middle-ear infections, GIT infections, urinary infections, respiratory infections and asthma, some childhood cancers, diarrhoeal diseases, juvenile
    diabetes, childhood obesity, allergies, eczema, SIDS
  • Health benefits for the mother – reduces the risk of breast cancer, ovarian cancer and osteoporosis; quicker return of uterus to pre-pregnancy size
  • Cost benefits
  • Breastmilk is sterile
  • Convenience and accessibility
160
Q

Define puerperal pyrexia?

A

fever ≥38°C in the first 14d postpartum, except the first day

161
Q

What are the causes of puerperal pyrexia?

HINT: B-5W

A
  • Breast: engorgement, mastitis
  • Wind: atelectasis, pneumonia
  • Water: UTI
  • Wound: episiotomy, C/S site infection
  • Walking: DVT, thrombophlebitis
  • Womb: endometritis
162
Q

Describe how to recognise, Ix, and Rx endometritis?

A
  • Endometritis – infection of the uterine myometrium and paraetrium
    • Clinical features – fever, chills, abdominal pain, uterine tenderness, foul-smelling discharge or lochia
    • Investigations – blood and genital cultures
    • Surgical prophylaxis – use cephazolin within the 60min before surgical incision
    • Management – depends on infection severity; oral antibiotics if well, IV with hospitalisation in moderate to severe cases (amoxy/ampicillin plus gentamycin plus metronidazole).
163
Q

Describe how to recognise, Ix, and Rx mastitis?

A
  • Mastitis – inflammation of mammary glands
    • Clinical features – fever, chills, myalgias, erythema, warmth, swelling, breast tenderness
    • Prevention – milk stasis implicated, treat with moist heat, massage, fluids, rest, proper positioning of the infant during nursing, nursing or manual expression of milk, analgesics
    • Management – Early treatment with antibiotics to prevent abscess formation; use di/flucoxacillin
164
Q

What are the causes of genital tract injury during bith?

A
  • Lacerations:
    • first degree: involves skin and vaginal mucosa but not underlying fascia and muscle
    • second degree: involves fascia and muscles of the perineal body but not the anal sphincter
    • third degree: involves the anal sphincter but does not extend through it
    • fourth degree: extends through the anal sphincter into the rectal mucosa
  • Episiotomy:
    • Incision in the perineal body at the time of delivery, essentially a controlled second decree laceration
    • Current evidence suggests letting perineum tear and then repair as needed
    • Indications – to relieve obstruction of the unyielding perineum, instrumental delivery
    • Complications – infection, haematoma, extension, fistula formation, incontinence
165
Q

What is the management of genital injuries during birth?

A
  • First and second degree tears – suture under local anaesthetic to increase healing and reduce pain
  • Third and fourth degree tears
    • Sphincter repaired under epidural or spinal anaesthesia in an operating theatre for visualisation and asepsis
    • Antiobiotics, laxatives and analgesia given, usually physiotherapy assessment
    • Sequelae – incontinence of flatus or urgency, occasionally frank incontinence, if ongoing incontinence problem vaginal birth in the future may make this worse (consider Caesarean)
166
Q

Describe post-partum blues?

A
  • Transient period of mild depression, mood instability, anxiety, decreased concentration, increased concern over own health and health of baby – considered to be normal emotions in the puerperium
  • Occurs in 50-80% of mothers; begins 2-4d postpartum, usually lasts 38hr, can last up to 10d
  • Does not require psychotropic medication, self-limiting (should resolve by 2wk)
  • Patient at increased risk of developing postpartum depression
167
Q

What is the DSM-V criteria for post partum depression?

A

DSM-V Criteria, Specifier for Major Depressive Episode with Peripartum Onset

  • This specifier can be applied to the current, or, if full criteria are not currently met for a major depressive episode, most recent episode of major depression if onset of mood symptoms occurs during pregnancy or in the 4 weeks following delivery
168
Q

What are the risk factors for post-partum depression?

A
  • Previous history of a mood disorder (postpartum or otherwise)
  • Psychosocial factors – stressful life events, unemployment, marital conflict, lack of social support, unwanted pregnancy, colicky or sick infant
169
Q

How is one assessed for postpartum depression?

A
  • Antenatal Risk Questionnaire (ANRQ) – to assess risk of PPD, components include:
    • past mental health history
    • past history of physical
    • sexual or emotional abuse
    • current level of supports
    • anxiety and obsessionality levels
    • stressors in the last year (including bereavement, separation etc.)
170
Q

What is the Rx for post partum depression?

A
  • Psychotherapy (CBT or IPT)
  • Short-term safety of maternal SSRIs for breastfeeding infants established, long-term effects unknown
  • If depression severe, consider ECT
171
Q

What is the consequence of psot-partum depression?

A
  • Child – increased risk of cognitive delay, insecure attachment, behavioural disorders
  • Treatment of mother improves outcome for child at 8m through increased mother-child interaction
172
Q

Define miscarriage, stillbirth and neonatal death.

A
  • Miscarriage – when a foetus is born with no signs of life before 24wk gestation
  • Stillbirth – when a foetus is delivered after 24wk completed gestation showing no signs of life
  • Neonatal death – death occurring within 28d of delivery
173
Q

What are the perinatal mortality rates in Australia and how do they rate to other countries?

A
  • The sum of stillbirths and early neonatal deaths per 1,000 total births
  • Has declined from >50.0 in the 1930s to 7.5 per 1,000 in 2008
  • The lowest rates are found in Scandinavia and the highest found in Bangladesh and Central Africa
174
Q

What are the causes of perinatal mortality?

A
  • unknown (20%)
  • preterm delivery (most common)
  • IUGR (>10%)
  • antepartum haemorrhage (~10%)
  • intrapartum still birth (9%)
  • major congenital abnormalities (10% of still births and 25% of neonatal mortality)
  • pre-eclampsia
  • infection
175
Q

Define maternal death, late maternal death, direct maternal death, indirect maternal death, incidental death/

A
  • Maternal death – death during pregnancy, or within 42d of its cessation, from any cause related to or aggravated by the pregnancy or its management, but not from accidental or incidental causes
  • Late maternal death – death related to pregnancy or management, 42d – 1y after birth
  • Direct death – result from obstetric complications of the pregnancy
  • Indirect death – result from previous or new disease, which was aggravated by pregnancy
  • Incidental death – would have happened irrespective of the pregnancy
176
Q

What are the maternal death rates and how do they compare to other countries?

A
  • ~11.4 per 100,000 pregnancies in the UK (2006-2008)
  • Deaths in less developed countries far higher, around 500 per 10,000 pregnancies in parts of Africa
177
Q

What are the causes of maternal mortality?

A
  • infection
  • thromboembolism
  • hypertensive disorders
  • cardiac disease
  • ectopic pregnancy and abortion
  • haemorrhage
  • neurological disease
  • psychiatric disease and suicide
178
Q

Neonatal goitres may be casues by what?

A
  • Maternal drugs such as carbimazole, iodine containing compounds
  • Pendred’s syndrome (associated with bilateral hearing loss and goitre)
  • Maternal Graves’ disease due to the passage of thyroid-stimulating hormone receptor antibodies across the placenta which will cause thyroid enlargement.